$\S 7.1$ 多元函数的基本概念

前置知识

邻域 , 内外点 , 边界点 , 聚点 , 开区域及闭区域 , 略

7.1.1 多元函数的定义

定义 7.1 设 $D$ 是 $n$ 维空间 $R_n$ 的非空子集 , $\rm R$ 为实数集 , 若按照某种对应法则 , 使得 对点集 内的每 个点 $x=(x1,x … , xn)$ , 都有唯一确定的 $uER$ 与之对应 , 则称对应法 是定义在 上的 元函数 , 记为-------------

与一元初等函数类似 , 多元初等函数是指能用一个算式表 示的多元函数 , 这个算式由不同变量的一元基本初等函数与常数经过有限次四则运算和复合而得到.

7.1.2 多元函数的极限

此本节重点讨论二元函数的极限 , 二元以上函数的极限以此类推.

定义 7.2 设 $n$ 元函数------------

多元函数极限与一元极限相同的性质有“唯一性” , “局部有界性” , “局部保号性” , “夹逼定理” , “Heine定理” , “极限四则运算法则”

⚠ 多元函数极限没有洛必达法则

多元函数趋近于 $(x_0,y_0)$ 的方式不可枚举. 因此说明极限不存在只要找到某个方向极限不存在 , 或两个方向极限不等.

7.1.3 多元函数的连续性

多元连续函数的性质与一元连续函数的性质完全类似. 根据多元函数极限的四则运算法则 , 可以证明: 多元连续函数的和、差、积仍为连续函数;多元连续函数的商在分母不为零处仍连续;多元连续函数的复合函数也是连续函数. 进一步可以得出如下结论: 一切多元初等函数在其定义区域内是连续的.

有界性与最大值最小值定理:

介值定理:

一致连续性定理:

$\S 7.2$ 偏导数与高阶偏导数

7.2.1 偏导数

定义 7.5

设二元函数 $z=f(x,y)$ 在点 $(x_0,y_0)$ 的某一邻域内有定义 , 则称极限

$$
\lim_{\Delta x \to 0} \frac{f(x_0 + \Delta x,y_0)-f(x_0,y_0)}{\Delta x}
$$

为对 $x$ 的偏导数. 记为 ${f}'_x(x_0,y_0)$ .

求在某一点偏导是否存在——直接代入偏导定义 , 看极限是否存在. (顺便求出偏导)

定理 7.1

如果函数 $z=f(x,y)$ 在点 $(x_0,y_0)$ 处二阶混合偏导连续 , 则必有

$$
{f}'_{xy}(x_0,y_0)={f}'_{yx}(x_0,y_0)
$$

初等函数的二阶混合偏导一定相等. 定理 7.1 对于二元以上的多元函数同样成立.

例 7.2.7 (Laplace) 拉普拉斯方程 (当结论)

函数 $u=\frac{1}{\sqrt{x^2+y^2+z^2}}$ , $u=z\arctan{\frac{x}{y}}$满足方程

$$
\frac{\partial^2u}{\partial x^2}+\frac{\partial^2u}{\partial y^2}+\frac{\partial^2u}{\partial z^2}=0
$$

换一种写法:

$$
{f}''_{xx}(x,y,z)+{f}''_{yy}(x,y,z)+{f}''_{zz}(x,y,z)=0
$$

$\S 7.3$ 全微分与高阶全微分

7.3.1 全微分的概念

$f(x_0+\Delta x,y_0)-f(x_0,y_0)={f}'_x(x_0,y_0)\Delta x+o(\Delta x)$ , 其中 ${f}'_x(x_0,y_0)\Delta x$ 称为 $z=f(x,y)$ 在点 $(x_0,y_0)$ 处对 $x$ 的偏微分.

$f(x_0,y_0+\Delta y)-f(x_0,y_0)={f}'_y(x_0,y_0)\Delta y+o(\Delta y)$ , 其中 ${f}'_y(x_0,y_0)\Delta x$ 称为 $z=f(x,y)$ 在点 $(x_0,y_0)$ 处对 $y$ 的偏微分.

定义 7.6

函数 $z=f(x,y)$ 在点 $(x_0,y_0)$ 处可微 $\Leftrightarrow$ $\frac{\Delta z-{d}z}{\rho} \to 0$ . 其中 $dz=A\;dx+B\;dy$ , $\rho=\sqrt{\Delta x^2+\Delta y^2}$ , $A={f}'_x(x_0+y_0)$ , $B={f}'_y(x_0+y_0)$ .

即:

$$
函数 z=f(x,y) 在点 (x_0,y_0) 处可微\Leftrightarrow \\ \lim_{\Delta x \to 0 \atop \Delta y \to 0} \frac{\big(f(x_0+\Delta x,y_0+\Delta y)-f(x_0,y_0)\big)-\big({f}'_x(x_0+y_0)\;\Delta x+{f}'_y(x_0+y_0)\;\Delta y\big)}{\sqrt{\Delta x^2+\Delta y^2}} \to 0 .
$$

函数 $z=f(x,y)$ 在点 $(x,y)$ 处的全微分

$$
\mathrm{d}z=\frac{\partial z}{\partial x} \Delta x + \frac{\partial z}{\partial y} \Delta y
$$

注意:由7.3.2可知 , 虽然函数可偏导的情况下能形式地写出全微分 , 但它不一定是全微分 , 需要通过定义去判断极限是否等于0.

由于自变量的微分即为自变量的增量 , 从而 $\mathrm{d}x=\Delta x$ , $\mathrm{d}y=\Delta y$ . 因此 函数 $z=f(x,y)$ 的全微分也可以写为 $\mathrm{d}z=\frac{\partial z}{\partial x} \mathrm{d}x+ \frac{\partial z}{\partial y} \mathrm{d}y$ .即两个偏微分之和 , 这一性质称为二元函数全微分的叠加原理.

7.3.2 连续、可偏导及可微的关系

屏幕截图 2023-05-28 112209

7.3.3 全微分的几何意义

7.3.4 全微分的计算与应用

全微分的计算

函数 $z=f(x,y)$ 在点 $(x,y)$ 处的全微分

$$
\mathrm{d}z=\frac{\partial z}{\partial x} \mathrm{d}x+ \frac{\partial z}{\partial y} \mathrm{d}y
$$

全微分的应用 · 近似计算

$$
{f(x_0+\Delta x,y_0+\Delta y)\approx f(x_0,y_0)+f_x(x_0+y_0)\;\Delta x+f_y(x_0+y_0)\;\Delta y}
$$

7.3.5 高阶全微分

$$
\mathrm{d}^nz=\mathrm{d}(\mathrm{d}^{n-1}z)
$$

$\S 7.2$ 多元复合函数的微分法

链式法则

一阶全微分形式不变性

隐函数存在定理及其求导法则

隐函数存在定理1

对于方程

$$
F(x_1,x_2,\cdots,x_n,y)=0,
$$

如果 $n+1$ 元函数 $F(x_1,x_2,\cdots,x_n,y)$ 满足条件:

  1. 在点 $P_0(x_1^0,x_2^0,\cdots,x_n^0,y^0)$ 的某一邻域内具有连续偏导数 $F_y$ , $F_{x_{i}}\ (i=1,2,\cdots,n)$ ;
  2. $F(x_1^0,x_2^0,\cdots,x_n^0,y^0)=0$ ;
  3. $F_y(x_1^0,x_2^0,\cdots,x_n^0,y^0)\ne 0$ ,

则在点 $P_0(x_1^0,x_2^0,\cdots,x_n^0,y^0)$ 的某一邻域内 , 方程

$$
F(x_1^0,x_2^0,\cdots,x_n^0,y^0)=0
$$

能唯一确定一个连续且具有连续偏导数的 $n$ 元函数 $y=f(x_1,x_2,\cdots,x_n)$ , 它满足条件

$$
F(x_1,x_2,\cdots,x_n,f(x_1,x_2,\cdots,x_n))\equiv 0\ ,\ y^0=f(x_1^0,x_2^0,\cdots,x_n^0),
$$

$$
\frac{\partial y}{\partial x_i}=-\frac{F_{x_i}}{F_y},\ \ i=1,2,\cdots,n.
$$

给定一个函数 $F=0$ , 问其是否在某一点处能唯一去认定一个有连续导数的隐函数:

  1. 求各个变量的偏导函数;
  2. 如果 $F$ 在该点处恒等于 $0$ (废话 , 点在 $F$ 上肯定满足 $F=0$ 啊) , 且该点处的某一变量的偏导不为 $0$ , 则对于这个变量存在隐函数.

隐函数存在定理2

对于方程组

$$
\left\{\begin{matrix}
F(x,y,u,v)=0, \\
G(x,y,u,v)=0,
\end{matrix}\right.
$$

如果函数 $F(x,y,u,v)$ 和 $G(x,y,u,v)$ 满足条件:

  1. 在点 $P_0(x_0,y_0,u_0,v_0)$ 的某一邻域内 $F$ 和 $G$ 对每个变量都具有连续偏导数;
  2. $F(x_0,y_0,u_0,v_0)=G(x_0,y_0,u_0,v_0)=0$ ;(废话)
  3. 在点 $P_0(x_0,y_0,u_0,v_0)$ 处的 $\mathrm{\mathbf{Jacobi}}$ 行列式
$$
J=\frac{\partial(F,G)}{\partial(u,v)}=\begin{vmatrix} {F}'_u & {F}'_v \\ {G}'_u & {G}'_v \end{vmatrix}\ne 0
$$

则方程组在点 $P_0(x_0,y_0,u_0,v_0)$ 的某一邻域内能唯一确定一个连续且具有连续偏导数的函数 $u=u(x,y)\ ,v=v(x,y)$ , 它们满足条件

$$
F(x,y,u(x,y),v(x,y))\equiv 0,\ \ G(x,y,u(x,y),v(x,y)) \equiv 0,\\u_0=u(x_0,y_0),\ \ v_0=v(x_0,y_0),
$$

$$
\frac{\partial {\color{red}u}}{\partial {\color{blue}x}}=-\frac{1}{J}\frac{\partial(F,G)}{\partial({\color{blue}x},{\color{magenta}v})}=-\frac{\begin{vmatrix} {F}'_{\color{blue}x} & {F}'_{\color{magenta}v} \\ {G}'_{\color{blue}x} & {G}'_{\color{magenta}v} \end{vmatrix}}{\begin{vmatrix} {F}'_{\color{red}u} & {F}'_{\color{magenta}v} \\ {G}'_{\color{red}u} & {G}'_{\color{magenta}v} \end{vmatrix}},
\\
\\
\frac{\partial {\color{magenta}v}}{\partial {\color{blue}x}}=-\frac{1}{J}\frac{\partial(F,G)}{\partial({\color{red}u},{\color{blue}x})}=-\frac{\begin{vmatrix} {F}'_{\color{red}u} & {F}'_{\color{blue}x} \\ {G}'_{\color{red}u} & {G}'_{\color{blue}x} \end{vmatrix}}{\begin{vmatrix} {F}'_{\color{red}u} & {F}'_{\color{magenta}v} \\ {G}'_{\color{red}u} & {G}'_{\color{magenta}v} \end{vmatrix}},
\\
\\
\frac{\partial {\color{red}u}}{\partial {\color{orange}y}}=-\frac{1}{J}\frac{\partial(F,G)}{\partial({\color{orange}y},{\color{magenta}v})}=-\frac{\begin{vmatrix} {F}'_{\color{orange}y} & {F}'_{\color{magenta}v} \\ {G}'_{\color{orange}y} & {G}'_{\color{magenta}v} \end{vmatrix}}{\begin{vmatrix} {F}'_{\color{red}u} & {F}'_{\color{magenta}v} \\ {G}'_{\color{red}u} & {G}'_{\color{magenta}v} \end{vmatrix}},
\\
\\
\frac{\partial {\color{magenta}v}}{\partial {\color{orange}y}}=-\frac{1}{J}\frac{\partial(F,G)}{\partial({\color{red}u},{\color{orange}y})}=-\frac{\begin{vmatrix} {F}'_{\color{red}u} & {F}'_{\color{orange}y} \\ {G}'_{\color{red}u} & {G}'_{\color{orange}y} \end{vmatrix}}{\begin{vmatrix} {F}'_{\color{red}u} & {F}'_{\color{magenta}v} \\ {G}'_{\color{red}u} & {G}'_{\color{magenta}v} \end{vmatrix}},
\\
$$

隐函数存在定理3

...≡(▔﹏▔)≡太复杂了不打了...

$\S 7.5$ 方向导数与梯度

7.5.1 方向导数

设 $l$ 是平面 $\mathbf{R}^2$ 上以 $P_0(x_0,y_0)$ 为始点的一条射线 , 其方向用向量 $\overrightarrow{l}$ 表示 , $e_l=(\cos{\alpha},\cos{\beta})$ 是与 $\overrightarrow{l}$ 同方向的单位向量 , 则 $\cos{\alpha},\cos{\beta}$ 是向量 $\overrightarrow{l}$ 的方向余弦.
定义 7.7 设函数 $z=f(x,y)$ 在点 $P_0(x_0,y_0)$ 的某一邻域内有定义 , $P(x_0+t\cos{\alpha},y_0+t\cos{\beta})$ 为 $l$ 上零一点 , 其中 $t$ 是 $P$ 到 $P_0$ 的距离 , 如果极限

$$
\lim_{t \to 0^+}\frac{f(x_0+t\cos\alpha,y_0+t\cos\beta)-f(x_0,y_0)}{t}
$$

存在 , 则称此极限为函数 $f(x,y)$ 在点 $P_0(x_0,y_0)$ 处沿方向 $\overrightarrow{l}$ 的方向导数 , 记作 $\frac{\partial f}{\partial l}\Big{|}_{(x_0,y_0)}.$

还可以表示为:

$$
\frac{\partial f}{\partial l}\Bigg{|}_{(x_0,y_0)}=\lim_{\rho \to 0^+}\frac{f(x_0+\Delta x,y_0+\Delta y)-f(x_0,y_0)}{\rho},\ \ \rho=\sqrt{(\Delta x)^2+(\Delta y)^2}.
$$

其中 $(\Delta x,\Delta y)$ 与 $\overrightarrow{l}$ 同向.

方向导数是朝一个方向的导数。偏导数则是沿着正负两个方向的导数,因此某点处沿任何方向的方向导数都存在也不能保证偏导数存在,因为有可能不相等。

定理 7.10 方向导数存在的充分条件及其计算公式

如果函数 $f(x,y)$ 在点 $P_0(x_0,y_0)$ 处可微,则函数在该点沿任一方向 $\overrightarrow l$ 的方向导数都存在,且有

$$
\frac{\partial f}{\partial l}\Bigg{|}_{(x_0,y_0)}={f}'_x(x_0,y_0)\cos\alpha+{f}'_y(x_0,y_0)\cos\beta,
$$

其中 $\cos\alpha,\cos\beta$ 是向量 $\overrightarrow l$ 的方向余弦。

7.5.2 多元函数的梯度

梯度,即某点处最大的方向导数。

$$
\mathbf{grad}\ f(x_0,y_0)=\nabla f(x_0,y_0)=\big(\ {f}'_x(x_0,y_0)\ ,\ {f}'_y(x_0,y_0)\ \big)^T
$$

更多元的函数的梯度同理。

有了梯度的定义,则函数 $f(x,y)$ 在点 $P_0(x_0.y_0)$ 处沿方向 $\overrightarrow l$ 的方向导数可以写成

$$
\frac{\partial f}{\partial l}\Bigg{|}_{(x_0,y_0)}={f}'_x(x_0,y_0)\cos\alpha+{f}'_y(x_0,y_0)\cos\beta=\mathbf{grad}\ f(x_0,y_0)\cdot \overrightarrow {e_l}=\left\| \mathbf{grad}\ f(x_0,y_0) \right\| \cos \theta.
$$

其中 $\left\| \mathbf{grad} f(x_0,y_0) \right\|$ 表示梯度的模, $\theta$ 是 $\overrightarrow{l}$ 和 $\mathbf{grad} f(x_0,y_0)$ 的夹角。

$\S 7.7$ 偏导数在几何中的应用

7.7.1 空间曲线的切线与法平面

空间曲线

设空间曲线 $\Gamma$ 的参数方程

$$
\left\{\begin{matrix}
x=\varphi(t) \\
y=\psi(t) \\
z=\omega(t)
\end{matrix}\right.\ ,(\alpha\le t \le \beta),
$$

其中 $\varphi(t),\psi(t),\omega(t)$ 可导且导函数不全为零。

则曲线 $\Gamma$ 在点 $M_0(x_0,y_0,z_0),t_0$ 处的切线方程

$$
\frac{x-x_0}{\varphi'(t_0)}=\frac{y-y_0}{\psi'(t_0)}=\frac{z-z_0}{\omega'(t_0)}.
$$

曲线 $\Gamma$ 在点 $M_0(x_0,y_0,z_0),t_0$ 处的法平面方程

$$
\varphi'(t_0)(x-x_0)+\psi'(t_0)(y-y_0)+\omega'(t_0)(z-z_0)=0.
$$

点 $M_0$ 处的切线的方向向量 $\overrightarrow{s}$ 切向量

$$
\overrightarrow{s}=(\varphi'(t_0),\psi'(t_0),\omega'(t_0)).
$$

$\overrightarrow s$ 的指向与参数 $t$ 增大时点 $M$ 移动的方向一致。

空间曲面

设曲面 $S$ 的方程为

$$
F(x,y,z)=0,
$$

函数 $F(x,y,z)$ 的偏导数在该点连续且不同时为零。

则曲面 $S$ 在点 $M_0(x_0,y_0,z_0)$ 处的切平面方程

$$
{F}'_x(x_0,y_0,z_0)(x-x_0)+{F}'_y(x_0,y_0,z_0)(y-y_0)+{F}'_z(x_0,y_0,z_0)(z-z_0)=0.
$$

曲面 $S$ 在点 $M_0$ 处的法线方程

$$
\frac{x-x_0}{{F}'_x(x_0,y_0,z_0)}=\frac{y-y_0}{{F}'_y(x_0,y_0,z_0)}=\frac{z-z_0}{{F}'_z(x_0,y_0,z_0)}.
$$

$\S 7.8 $ 多元函数的极值

7.8.1 无条件极值

多元函数的极值

极大值,极小值,极大值点,极小值点的定义,见书 定义 7.14,与一元函数的定义类似。

定理 7.1.4 ( 二元函数极值的必要条件 ) 若函数 $z=f(x,y)$ 在点 $(x_0,y_0)$ 处可偏导,且 $(x_0,y_0)$ 为 $f(x,y)$ 的极值点,则有

$$
{f}'_x(x_0,y_0)=0,\ \ \ {f}'_y(x_0,y_0)=0.
$$

换句话说,极值点处的梯度为零向量。

二元函数极值的充分条件

若函数 $z=f(x,y)$ 在点 $(x_0,y_0)$ 的某个邻域内有二阶连续偏导数,且 $(x_0,y_0)$ 为 $f(x,y)$ 的驻点,记

$$
A={f}''_{xx}(x_0,y_0),\ \ \ B={f}''_{xy}(x_0,y_0),\ \ \ C={f}''_{yy}(x_0,y_0),\ \ \
$$

  1. 若 $AC-B^2>0$,则 $f(x_0,y_0)$ 是极值,且当 $A<0$ 时为极大值,$A>0$ 时为极小值。
  2. 若 $AC-B^2<0$,则 $f(x_0,y_0)$ 一定不是极值。
  3. 若 $AC-B^2=0$,则该方法失效。

多元函数极值的充分条件(了解,有学科交叉)

略略略...

7.8.2 多元函数的最值

如何求多元函数的最值?与一元函数思路一样,最值一定是在极值处或者边界处出现。找极值的方法在上一节介绍过了,找边界处的最值的方法,需要用到7.8.3节的条件极值 Lagrange 乘数法。

7.8.3 条件极值和 Lagrange 乘数法

条件极值

条件极值,即对自变量附加约束条件的极值。例如,计算函数 $f(x,y,z)$ 在某一约束条件 $\varphi (x,y,z)=0$ 下的最小值,就是条件极值。称 $f(x,y,z)$ 为目标函数,称 $\varphi(x,y,z)=0$ 为约束条件

Lagrange 乘数法

若想要求函数 $f(x,y,z)$ 在约束条件 $\varphi(x,y,z)=0$ 下的可能极值点,首先构造 $\mathrm{Lagrange}$ 函数

$$
L(x,y,z,\lambda)=f(x,y,z)+\lambda\varphi(x,y,z),
$$

其中参数 $\lambda$ 为 $\mathbf{Lagrange}$ 乘子,再求 $L$ 各个变量的一阶偏导数,并令其为零,得到

$$
\left\{\begin{matrix}
L_x={f}'_x(x,y,z)+\lambda{\varphi}'_x(x_0,y_0,z_0)=0 \hfill\\
L_y={f}'_y(x,y,z)+\lambda{\varphi}'_y(x_0,y_0,z_0)=0 \hfill\\
L_z={f}'_z(x,y,z)+\lambda{\varphi}'_z(x_0,y_0,z_0)=0 \hfill\\
L_\lambda={\varphi}(x_0,y_0,z_0)=0 \hfill\\
\end{matrix}\right.
$$

解上述方程组得 $x_0,y_0,z_0$ 及 $\lambda$ ,则点 $(x_0,y_0,z_0)$ 就是在这一约束条件下的可能极值点。

如果有更多的约束条件比如 $\varphi=0,\ \ \psi=0$ ,那么就取两个 $\mathrm{Lagrange}$ 乘子 $\lambda,\mu$ ,则 $L=f+\lambda\varphi+\mu\psi$(简写了).


end. 终于写完了 இ௰இ